Domain of definition of Laplace Operator on L^2How to construct a scalar differential operator having the same spectrum as a non-scalar differential operator exploiting symmetries?Why is this operator compact?Stokes operator without Dirichlet boundary conditionDomain of $A^1/2$ on $L^2(mathbbT^2)$Characterisation of Sobolev spaces using eigenvalues of laplacianThe inverse of Laplacian operator for different ordersDomain of the Stokes operatorDistribution that vanishes against approximated delta is zero$(ipartial_t)^frac12 e^itDelta f = (-Delta )^frac12 e^itDelta f$?One-parameter unitary group preserving invariant domain of infinitesimal generator

Domain of definition of Laplace Operator on L^2


How to construct a scalar differential operator having the same spectrum as a non-scalar differential operator exploiting symmetries?Why is this operator compact?Stokes operator without Dirichlet boundary conditionDomain of $A^1/2$ on $L^2(mathbbT^2)$Characterisation of Sobolev spaces using eigenvalues of laplacianThe inverse of Laplacian operator for different ordersDomain of the Stokes operatorDistribution that vanishes against approximated delta is zero$(ipartial_t)^frac12 e^itDelta f = (-Delta )^frac12 e^itDelta f$?One-parameter unitary group preserving invariant domain of infinitesimal generator













2












$begingroup$


I'm trying to combine two ways of looking at the Laplacian $Delta$ on $mathbb R^n$ (and on other domains).



Firstly, it is well known that this operator is essentially self-adjoint on $C_c^infty(mathbb R^n)$. Secondly, I know that for $f,g in C^infty_c(mathbb R^n)$ it holds that $langle u, Delta u rangle = langle nabla u, nabla v rangle $, where the inner-product is the $L^2$ one.



Now the domain $mathcal D (Delta) subset L^2$ on which the Laplacian is self-adjoint must contain $C_c^infty (mathbb R^n)$, but I can't seem to find much more information on this. By the previous identity I feel like $Delta u in L^2$ should imply that $u in H^1(mathbb R^n)$, but I can't seem to find a way to prove this. I know it is possible to prove that $mathcal D(Delta) subset H^2(mathbb R^n$) via elliptic regularity etc, but it seems to me like there "should" be an elementary argument for showing $mathcal D(Delta) subset H^1$ (preferably via the non-Fourier-transform characterisation of weak derivatives) but I can't seem to find it anywhere.



Edit: To clarify my question: I am looking for a simple proof of $Delta u = f in L^2$ implies $u in H^1$ where $Delta u$ is defined as the self-adjoint extension of the Laplacian $Delta$ on $C^infty_c$, preferrably one which also works on domains other than $mathbb R^n$.










share|cite|improve this question











$endgroup$







  • 1




    $begingroup$
    Are you familiar with the Friedrichs extension theorem?
    $endgroup$
    – Nate Eldredge
    9 hours ago










  • $begingroup$
    I am not, but this seems like it would apply, thank you for the reference!
    $endgroup$
    – Nathanael Schilling
    9 hours ago










  • $begingroup$
    You want to state the question a little differently for other domains $Omega$, because the Laplacian is typically not essentially self-adjoint on $C^infty_c(Omega)$ in that case.
    $endgroup$
    – Nate Eldredge
    2 hours ago
















2












$begingroup$


I'm trying to combine two ways of looking at the Laplacian $Delta$ on $mathbb R^n$ (and on other domains).



Firstly, it is well known that this operator is essentially self-adjoint on $C_c^infty(mathbb R^n)$. Secondly, I know that for $f,g in C^infty_c(mathbb R^n)$ it holds that $langle u, Delta u rangle = langle nabla u, nabla v rangle $, where the inner-product is the $L^2$ one.



Now the domain $mathcal D (Delta) subset L^2$ on which the Laplacian is self-adjoint must contain $C_c^infty (mathbb R^n)$, but I can't seem to find much more information on this. By the previous identity I feel like $Delta u in L^2$ should imply that $u in H^1(mathbb R^n)$, but I can't seem to find a way to prove this. I know it is possible to prove that $mathcal D(Delta) subset H^2(mathbb R^n$) via elliptic regularity etc, but it seems to me like there "should" be an elementary argument for showing $mathcal D(Delta) subset H^1$ (preferably via the non-Fourier-transform characterisation of weak derivatives) but I can't seem to find it anywhere.



Edit: To clarify my question: I am looking for a simple proof of $Delta u = f in L^2$ implies $u in H^1$ where $Delta u$ is defined as the self-adjoint extension of the Laplacian $Delta$ on $C^infty_c$, preferrably one which also works on domains other than $mathbb R^n$.










share|cite|improve this question











$endgroup$







  • 1




    $begingroup$
    Are you familiar with the Friedrichs extension theorem?
    $endgroup$
    – Nate Eldredge
    9 hours ago










  • $begingroup$
    I am not, but this seems like it would apply, thank you for the reference!
    $endgroup$
    – Nathanael Schilling
    9 hours ago










  • $begingroup$
    You want to state the question a little differently for other domains $Omega$, because the Laplacian is typically not essentially self-adjoint on $C^infty_c(Omega)$ in that case.
    $endgroup$
    – Nate Eldredge
    2 hours ago














2












2








2





$begingroup$


I'm trying to combine two ways of looking at the Laplacian $Delta$ on $mathbb R^n$ (and on other domains).



Firstly, it is well known that this operator is essentially self-adjoint on $C_c^infty(mathbb R^n)$. Secondly, I know that for $f,g in C^infty_c(mathbb R^n)$ it holds that $langle u, Delta u rangle = langle nabla u, nabla v rangle $, where the inner-product is the $L^2$ one.



Now the domain $mathcal D (Delta) subset L^2$ on which the Laplacian is self-adjoint must contain $C_c^infty (mathbb R^n)$, but I can't seem to find much more information on this. By the previous identity I feel like $Delta u in L^2$ should imply that $u in H^1(mathbb R^n)$, but I can't seem to find a way to prove this. I know it is possible to prove that $mathcal D(Delta) subset H^2(mathbb R^n$) via elliptic regularity etc, but it seems to me like there "should" be an elementary argument for showing $mathcal D(Delta) subset H^1$ (preferably via the non-Fourier-transform characterisation of weak derivatives) but I can't seem to find it anywhere.



Edit: To clarify my question: I am looking for a simple proof of $Delta u = f in L^2$ implies $u in H^1$ where $Delta u$ is defined as the self-adjoint extension of the Laplacian $Delta$ on $C^infty_c$, preferrably one which also works on domains other than $mathbb R^n$.










share|cite|improve this question











$endgroup$




I'm trying to combine two ways of looking at the Laplacian $Delta$ on $mathbb R^n$ (and on other domains).



Firstly, it is well known that this operator is essentially self-adjoint on $C_c^infty(mathbb R^n)$. Secondly, I know that for $f,g in C^infty_c(mathbb R^n)$ it holds that $langle u, Delta u rangle = langle nabla u, nabla v rangle $, where the inner-product is the $L^2$ one.



Now the domain $mathcal D (Delta) subset L^2$ on which the Laplacian is self-adjoint must contain $C_c^infty (mathbb R^n)$, but I can't seem to find much more information on this. By the previous identity I feel like $Delta u in L^2$ should imply that $u in H^1(mathbb R^n)$, but I can't seem to find a way to prove this. I know it is possible to prove that $mathcal D(Delta) subset H^2(mathbb R^n$) via elliptic regularity etc, but it seems to me like there "should" be an elementary argument for showing $mathcal D(Delta) subset H^1$ (preferably via the non-Fourier-transform characterisation of weak derivatives) but I can't seem to find it anywhere.



Edit: To clarify my question: I am looking for a simple proof of $Delta u = f in L^2$ implies $u in H^1$ where $Delta u$ is defined as the self-adjoint extension of the Laplacian $Delta$ on $C^infty_c$, preferrably one which also works on domains other than $mathbb R^n$.







fa.functional-analysis ap.analysis-of-pdes






share|cite|improve this question















share|cite|improve this question













share|cite|improve this question




share|cite|improve this question








edited 9 hours ago







Nathanael Schilling

















asked 11 hours ago









Nathanael SchillingNathanael Schilling

214 bronze badges




214 bronze badges







  • 1




    $begingroup$
    Are you familiar with the Friedrichs extension theorem?
    $endgroup$
    – Nate Eldredge
    9 hours ago










  • $begingroup$
    I am not, but this seems like it would apply, thank you for the reference!
    $endgroup$
    – Nathanael Schilling
    9 hours ago










  • $begingroup$
    You want to state the question a little differently for other domains $Omega$, because the Laplacian is typically not essentially self-adjoint on $C^infty_c(Omega)$ in that case.
    $endgroup$
    – Nate Eldredge
    2 hours ago













  • 1




    $begingroup$
    Are you familiar with the Friedrichs extension theorem?
    $endgroup$
    – Nate Eldredge
    9 hours ago










  • $begingroup$
    I am not, but this seems like it would apply, thank you for the reference!
    $endgroup$
    – Nathanael Schilling
    9 hours ago










  • $begingroup$
    You want to state the question a little differently for other domains $Omega$, because the Laplacian is typically not essentially self-adjoint on $C^infty_c(Omega)$ in that case.
    $endgroup$
    – Nate Eldredge
    2 hours ago








1




1




$begingroup$
Are you familiar with the Friedrichs extension theorem?
$endgroup$
– Nate Eldredge
9 hours ago




$begingroup$
Are you familiar with the Friedrichs extension theorem?
$endgroup$
– Nate Eldredge
9 hours ago












$begingroup$
I am not, but this seems like it would apply, thank you for the reference!
$endgroup$
– Nathanael Schilling
9 hours ago




$begingroup$
I am not, but this seems like it would apply, thank you for the reference!
$endgroup$
– Nathanael Schilling
9 hours ago












$begingroup$
You want to state the question a little differently for other domains $Omega$, because the Laplacian is typically not essentially self-adjoint on $C^infty_c(Omega)$ in that case.
$endgroup$
– Nate Eldredge
2 hours ago





$begingroup$
You want to state the question a little differently for other domains $Omega$, because the Laplacian is typically not essentially self-adjoint on $C^infty_c(Omega)$ in that case.
$endgroup$
– Nate Eldredge
2 hours ago











2 Answers
2






active

oldest

votes


















3












$begingroup$

The Laplace operator is essentially self-adjoint: define
$$
mathcal D(∆)=uin L^2, ∆uin L^2=H^2.
$$

Then for $u,vin mathcal D(∆)$, $langle ∆ u,vrangle=langle u,∆vrangle$: to prove this, consider $lim u_k=u, lim v_k=v text in $H^2$$ with $u_k, v_k$ smooth compactly supported. Then you have
$$
langle ∆ u,vrangle=lim_klangle ∆ u,v_krangle=lim_klim_llangle ∆ u_l,v_krangle=lim_klim_llangle u_l,∆v_krangle=lim_klangle u,∆v_krangle=langle u,∆vrangle, textqed.
$$

Moreover you define for the symmetric operator $∆$,
$$
mathcal D(∆^*)=vin L^2,exists C, forall uin mathcal D(∆),
vertlangle ∆u,vranglevertle CVert uVert_L^2.
$$

Then as for all symmetric operators we have $mathcal D(∆^*)supset mathcal D(∆)$. To get self-adjointness, you must prove $mathcal D(∆^*)subset mathcal D(∆)$. Let $vin mathcal D(∆^*)$. Let $u$ be a smooth compactly supported function: then we have
$$
vertlangle ∆u,vranglevertle CVert uVert_L^2, quad texti.e. quad
vertlangle u,∆vranglevertle CVert uVert_L^2,
$$

proving that $∆ vin L^2$ and since $vin L^2$, the ellipticity of the Laplace operator implies that $uin H^2=mathcal D(∆)$. The above argument works for any symmetric elliptic operator.






share|cite|improve this answer









$endgroup$












  • $begingroup$
    I know the Laplacian is essentially self-adjoint, I was maybe a bit unclear but my question was why its domain of definition is a subset of $H^1$, not why the operator is essentially self-adjoint. The definition of $H^2$ you give isn't the standard one (existence of all weak partial derivatives of order 1 and 2 in $L^2$), I would like to know why $mathcal D(Delta) subset H^1$ with the standard definition.
    $endgroup$
    – Nathanael Schilling
    10 hours ago











  • $begingroup$
    Maybe you might ask yourself what the relation of $H^2$ and $H^1$ is.
    $endgroup$
    – Fabian Wirth
    10 hours ago










  • $begingroup$
    I think I misunderstood the post above in that I thought it was a definition for $H^2$, whereas the definition was for $mathcal D(Delta)$ in terms of $H^2$. Nevertheless, my question still stands as the point I was asking about is contained in the reference to the ellipticity of the Laplace operator; which is nontrivial on general domains as far as I remember. This is why I would like a simple proof of the weaker statement that the domain is contained in $H^1$.
    $endgroup$
    – Nathanael Schilling
    9 hours ago











  • $begingroup$
    In this situation, essential self-adjointness is exactly that the domain of the unique self-adjoint extension is $H^2$. (More generally, for semi-bounded operators, we can define analogous $H^2$, Friedrichs' extensions, and so on.)
    $endgroup$
    – paul garrett
    4 hours ago


















1












$begingroup$

Maybe this is the argument you're looking for.



Suppose you know that $Delta$ is essentially self-adjoint on $C^infty_c(mathbbR^n)$. This means $C^infty_c$ is a core for $Delta$, so for any $u in D(Delta)$, there is a sequence $u_n in C^infty_c$ such that $u_n to u$ and $Delta u_n to Delta u$ in $L^2$. In particular, we have
$$|langle Delta(u_n - u_m), u_n - u_m rangle_L^2 | le |Delta u_n - Delta u_m|_L^2 |u_n - u_m|_L^2 to 0$$
using Cauchy-Schwarz. On the other hand, since $u_n, u_m in C^infty_c$, we are perfectly justified in integrating by parts to say that $langle Delta(u_n - u_m), u_n - u_m rangle = |nabla (u_n - u_m)|^2_L^2$. So, since $|u_n - u_m|_L^2 to 0$ and $|nabla (u_n - u_m)|_L^2 to 0$, we have shown that $|u_n - u_m|_H^1 to 0$; in other words, $u_n$ is Cauchy in $H^1$. Now $H^1$ is a Hilbert space, so $u_n$ converges in $H^1$ to some $v in H^1$. But we already know $u_n to u$ in $L^2$, which is weaker than $H^1$ convergence, so we must have $u = v$, thus $u in H^1$.






share|cite|improve this answer









$endgroup$















    Your Answer








    StackExchange.ready(function()
    var channelOptions =
    tags: "".split(" "),
    id: "504"
    ;
    initTagRenderer("".split(" "), "".split(" "), channelOptions);

    StackExchange.using("externalEditor", function()
    // Have to fire editor after snippets, if snippets enabled
    if (StackExchange.settings.snippets.snippetsEnabled)
    StackExchange.using("snippets", function()
    createEditor();
    );

    else
    createEditor();

    );

    function createEditor()
    StackExchange.prepareEditor(
    heartbeatType: 'answer',
    autoActivateHeartbeat: false,
    convertImagesToLinks: true,
    noModals: true,
    showLowRepImageUploadWarning: true,
    reputationToPostImages: 10,
    bindNavPrevention: true,
    postfix: "",
    imageUploader:
    brandingHtml: "Powered by u003ca class="icon-imgur-white" href="https://imgur.com/"u003eu003c/au003e",
    contentPolicyHtml: "User contributions licensed under u003ca href="https://creativecommons.org/licenses/by-sa/3.0/"u003ecc by-sa 3.0 with attribution requiredu003c/au003e u003ca href="https://stackoverflow.com/legal/content-policy"u003e(content policy)u003c/au003e",
    allowUrls: true
    ,
    noCode: true, onDemand: true,
    discardSelector: ".discard-answer"
    ,immediatelyShowMarkdownHelp:true
    );



    );













    draft saved

    draft discarded


















    StackExchange.ready(
    function ()
    StackExchange.openid.initPostLogin('.new-post-login', 'https%3a%2f%2fmathoverflow.net%2fquestions%2f335952%2fdomain-of-definition-of-laplace-operator-on-l2%23new-answer', 'question_page');

    );

    Post as a guest















    Required, but never shown

























    2 Answers
    2






    active

    oldest

    votes








    2 Answers
    2






    active

    oldest

    votes









    active

    oldest

    votes






    active

    oldest

    votes









    3












    $begingroup$

    The Laplace operator is essentially self-adjoint: define
    $$
    mathcal D(∆)=uin L^2, ∆uin L^2=H^2.
    $$

    Then for $u,vin mathcal D(∆)$, $langle ∆ u,vrangle=langle u,∆vrangle$: to prove this, consider $lim u_k=u, lim v_k=v text in $H^2$$ with $u_k, v_k$ smooth compactly supported. Then you have
    $$
    langle ∆ u,vrangle=lim_klangle ∆ u,v_krangle=lim_klim_llangle ∆ u_l,v_krangle=lim_klim_llangle u_l,∆v_krangle=lim_klangle u,∆v_krangle=langle u,∆vrangle, textqed.
    $$

    Moreover you define for the symmetric operator $∆$,
    $$
    mathcal D(∆^*)=vin L^2,exists C, forall uin mathcal D(∆),
    vertlangle ∆u,vranglevertle CVert uVert_L^2.
    $$

    Then as for all symmetric operators we have $mathcal D(∆^*)supset mathcal D(∆)$. To get self-adjointness, you must prove $mathcal D(∆^*)subset mathcal D(∆)$. Let $vin mathcal D(∆^*)$. Let $u$ be a smooth compactly supported function: then we have
    $$
    vertlangle ∆u,vranglevertle CVert uVert_L^2, quad texti.e. quad
    vertlangle u,∆vranglevertle CVert uVert_L^2,
    $$

    proving that $∆ vin L^2$ and since $vin L^2$, the ellipticity of the Laplace operator implies that $uin H^2=mathcal D(∆)$. The above argument works for any symmetric elliptic operator.






    share|cite|improve this answer









    $endgroup$












    • $begingroup$
      I know the Laplacian is essentially self-adjoint, I was maybe a bit unclear but my question was why its domain of definition is a subset of $H^1$, not why the operator is essentially self-adjoint. The definition of $H^2$ you give isn't the standard one (existence of all weak partial derivatives of order 1 and 2 in $L^2$), I would like to know why $mathcal D(Delta) subset H^1$ with the standard definition.
      $endgroup$
      – Nathanael Schilling
      10 hours ago











    • $begingroup$
      Maybe you might ask yourself what the relation of $H^2$ and $H^1$ is.
      $endgroup$
      – Fabian Wirth
      10 hours ago










    • $begingroup$
      I think I misunderstood the post above in that I thought it was a definition for $H^2$, whereas the definition was for $mathcal D(Delta)$ in terms of $H^2$. Nevertheless, my question still stands as the point I was asking about is contained in the reference to the ellipticity of the Laplace operator; which is nontrivial on general domains as far as I remember. This is why I would like a simple proof of the weaker statement that the domain is contained in $H^1$.
      $endgroup$
      – Nathanael Schilling
      9 hours ago











    • $begingroup$
      In this situation, essential self-adjointness is exactly that the domain of the unique self-adjoint extension is $H^2$. (More generally, for semi-bounded operators, we can define analogous $H^2$, Friedrichs' extensions, and so on.)
      $endgroup$
      – paul garrett
      4 hours ago















    3












    $begingroup$

    The Laplace operator is essentially self-adjoint: define
    $$
    mathcal D(∆)=uin L^2, ∆uin L^2=H^2.
    $$

    Then for $u,vin mathcal D(∆)$, $langle ∆ u,vrangle=langle u,∆vrangle$: to prove this, consider $lim u_k=u, lim v_k=v text in $H^2$$ with $u_k, v_k$ smooth compactly supported. Then you have
    $$
    langle ∆ u,vrangle=lim_klangle ∆ u,v_krangle=lim_klim_llangle ∆ u_l,v_krangle=lim_klim_llangle u_l,∆v_krangle=lim_klangle u,∆v_krangle=langle u,∆vrangle, textqed.
    $$

    Moreover you define for the symmetric operator $∆$,
    $$
    mathcal D(∆^*)=vin L^2,exists C, forall uin mathcal D(∆),
    vertlangle ∆u,vranglevertle CVert uVert_L^2.
    $$

    Then as for all symmetric operators we have $mathcal D(∆^*)supset mathcal D(∆)$. To get self-adjointness, you must prove $mathcal D(∆^*)subset mathcal D(∆)$. Let $vin mathcal D(∆^*)$. Let $u$ be a smooth compactly supported function: then we have
    $$
    vertlangle ∆u,vranglevertle CVert uVert_L^2, quad texti.e. quad
    vertlangle u,∆vranglevertle CVert uVert_L^2,
    $$

    proving that $∆ vin L^2$ and since $vin L^2$, the ellipticity of the Laplace operator implies that $uin H^2=mathcal D(∆)$. The above argument works for any symmetric elliptic operator.






    share|cite|improve this answer









    $endgroup$












    • $begingroup$
      I know the Laplacian is essentially self-adjoint, I was maybe a bit unclear but my question was why its domain of definition is a subset of $H^1$, not why the operator is essentially self-adjoint. The definition of $H^2$ you give isn't the standard one (existence of all weak partial derivatives of order 1 and 2 in $L^2$), I would like to know why $mathcal D(Delta) subset H^1$ with the standard definition.
      $endgroup$
      – Nathanael Schilling
      10 hours ago











    • $begingroup$
      Maybe you might ask yourself what the relation of $H^2$ and $H^1$ is.
      $endgroup$
      – Fabian Wirth
      10 hours ago










    • $begingroup$
      I think I misunderstood the post above in that I thought it was a definition for $H^2$, whereas the definition was for $mathcal D(Delta)$ in terms of $H^2$. Nevertheless, my question still stands as the point I was asking about is contained in the reference to the ellipticity of the Laplace operator; which is nontrivial on general domains as far as I remember. This is why I would like a simple proof of the weaker statement that the domain is contained in $H^1$.
      $endgroup$
      – Nathanael Schilling
      9 hours ago











    • $begingroup$
      In this situation, essential self-adjointness is exactly that the domain of the unique self-adjoint extension is $H^2$. (More generally, for semi-bounded operators, we can define analogous $H^2$, Friedrichs' extensions, and so on.)
      $endgroup$
      – paul garrett
      4 hours ago













    3












    3








    3





    $begingroup$

    The Laplace operator is essentially self-adjoint: define
    $$
    mathcal D(∆)=uin L^2, ∆uin L^2=H^2.
    $$

    Then for $u,vin mathcal D(∆)$, $langle ∆ u,vrangle=langle u,∆vrangle$: to prove this, consider $lim u_k=u, lim v_k=v text in $H^2$$ with $u_k, v_k$ smooth compactly supported. Then you have
    $$
    langle ∆ u,vrangle=lim_klangle ∆ u,v_krangle=lim_klim_llangle ∆ u_l,v_krangle=lim_klim_llangle u_l,∆v_krangle=lim_klangle u,∆v_krangle=langle u,∆vrangle, textqed.
    $$

    Moreover you define for the symmetric operator $∆$,
    $$
    mathcal D(∆^*)=vin L^2,exists C, forall uin mathcal D(∆),
    vertlangle ∆u,vranglevertle CVert uVert_L^2.
    $$

    Then as for all symmetric operators we have $mathcal D(∆^*)supset mathcal D(∆)$. To get self-adjointness, you must prove $mathcal D(∆^*)subset mathcal D(∆)$. Let $vin mathcal D(∆^*)$. Let $u$ be a smooth compactly supported function: then we have
    $$
    vertlangle ∆u,vranglevertle CVert uVert_L^2, quad texti.e. quad
    vertlangle u,∆vranglevertle CVert uVert_L^2,
    $$

    proving that $∆ vin L^2$ and since $vin L^2$, the ellipticity of the Laplace operator implies that $uin H^2=mathcal D(∆)$. The above argument works for any symmetric elliptic operator.






    share|cite|improve this answer









    $endgroup$



    The Laplace operator is essentially self-adjoint: define
    $$
    mathcal D(∆)=uin L^2, ∆uin L^2=H^2.
    $$

    Then for $u,vin mathcal D(∆)$, $langle ∆ u,vrangle=langle u,∆vrangle$: to prove this, consider $lim u_k=u, lim v_k=v text in $H^2$$ with $u_k, v_k$ smooth compactly supported. Then you have
    $$
    langle ∆ u,vrangle=lim_klangle ∆ u,v_krangle=lim_klim_llangle ∆ u_l,v_krangle=lim_klim_llangle u_l,∆v_krangle=lim_klangle u,∆v_krangle=langle u,∆vrangle, textqed.
    $$

    Moreover you define for the symmetric operator $∆$,
    $$
    mathcal D(∆^*)=vin L^2,exists C, forall uin mathcal D(∆),
    vertlangle ∆u,vranglevertle CVert uVert_L^2.
    $$

    Then as for all symmetric operators we have $mathcal D(∆^*)supset mathcal D(∆)$. To get self-adjointness, you must prove $mathcal D(∆^*)subset mathcal D(∆)$. Let $vin mathcal D(∆^*)$. Let $u$ be a smooth compactly supported function: then we have
    $$
    vertlangle ∆u,vranglevertle CVert uVert_L^2, quad texti.e. quad
    vertlangle u,∆vranglevertle CVert uVert_L^2,
    $$

    proving that $∆ vin L^2$ and since $vin L^2$, the ellipticity of the Laplace operator implies that $uin H^2=mathcal D(∆)$. The above argument works for any symmetric elliptic operator.







    share|cite|improve this answer












    share|cite|improve this answer



    share|cite|improve this answer










    answered 10 hours ago









    BazinBazin

    8,75915 silver badges39 bronze badges




    8,75915 silver badges39 bronze badges











    • $begingroup$
      I know the Laplacian is essentially self-adjoint, I was maybe a bit unclear but my question was why its domain of definition is a subset of $H^1$, not why the operator is essentially self-adjoint. The definition of $H^2$ you give isn't the standard one (existence of all weak partial derivatives of order 1 and 2 in $L^2$), I would like to know why $mathcal D(Delta) subset H^1$ with the standard definition.
      $endgroup$
      – Nathanael Schilling
      10 hours ago











    • $begingroup$
      Maybe you might ask yourself what the relation of $H^2$ and $H^1$ is.
      $endgroup$
      – Fabian Wirth
      10 hours ago










    • $begingroup$
      I think I misunderstood the post above in that I thought it was a definition for $H^2$, whereas the definition was for $mathcal D(Delta)$ in terms of $H^2$. Nevertheless, my question still stands as the point I was asking about is contained in the reference to the ellipticity of the Laplace operator; which is nontrivial on general domains as far as I remember. This is why I would like a simple proof of the weaker statement that the domain is contained in $H^1$.
      $endgroup$
      – Nathanael Schilling
      9 hours ago











    • $begingroup$
      In this situation, essential self-adjointness is exactly that the domain of the unique self-adjoint extension is $H^2$. (More generally, for semi-bounded operators, we can define analogous $H^2$, Friedrichs' extensions, and so on.)
      $endgroup$
      – paul garrett
      4 hours ago
















    • $begingroup$
      I know the Laplacian is essentially self-adjoint, I was maybe a bit unclear but my question was why its domain of definition is a subset of $H^1$, not why the operator is essentially self-adjoint. The definition of $H^2$ you give isn't the standard one (existence of all weak partial derivatives of order 1 and 2 in $L^2$), I would like to know why $mathcal D(Delta) subset H^1$ with the standard definition.
      $endgroup$
      – Nathanael Schilling
      10 hours ago











    • $begingroup$
      Maybe you might ask yourself what the relation of $H^2$ and $H^1$ is.
      $endgroup$
      – Fabian Wirth
      10 hours ago










    • $begingroup$
      I think I misunderstood the post above in that I thought it was a definition for $H^2$, whereas the definition was for $mathcal D(Delta)$ in terms of $H^2$. Nevertheless, my question still stands as the point I was asking about is contained in the reference to the ellipticity of the Laplace operator; which is nontrivial on general domains as far as I remember. This is why I would like a simple proof of the weaker statement that the domain is contained in $H^1$.
      $endgroup$
      – Nathanael Schilling
      9 hours ago











    • $begingroup$
      In this situation, essential self-adjointness is exactly that the domain of the unique self-adjoint extension is $H^2$. (More generally, for semi-bounded operators, we can define analogous $H^2$, Friedrichs' extensions, and so on.)
      $endgroup$
      – paul garrett
      4 hours ago















    $begingroup$
    I know the Laplacian is essentially self-adjoint, I was maybe a bit unclear but my question was why its domain of definition is a subset of $H^1$, not why the operator is essentially self-adjoint. The definition of $H^2$ you give isn't the standard one (existence of all weak partial derivatives of order 1 and 2 in $L^2$), I would like to know why $mathcal D(Delta) subset H^1$ with the standard definition.
    $endgroup$
    – Nathanael Schilling
    10 hours ago





    $begingroup$
    I know the Laplacian is essentially self-adjoint, I was maybe a bit unclear but my question was why its domain of definition is a subset of $H^1$, not why the operator is essentially self-adjoint. The definition of $H^2$ you give isn't the standard one (existence of all weak partial derivatives of order 1 and 2 in $L^2$), I would like to know why $mathcal D(Delta) subset H^1$ with the standard definition.
    $endgroup$
    – Nathanael Schilling
    10 hours ago













    $begingroup$
    Maybe you might ask yourself what the relation of $H^2$ and $H^1$ is.
    $endgroup$
    – Fabian Wirth
    10 hours ago




    $begingroup$
    Maybe you might ask yourself what the relation of $H^2$ and $H^1$ is.
    $endgroup$
    – Fabian Wirth
    10 hours ago












    $begingroup$
    I think I misunderstood the post above in that I thought it was a definition for $H^2$, whereas the definition was for $mathcal D(Delta)$ in terms of $H^2$. Nevertheless, my question still stands as the point I was asking about is contained in the reference to the ellipticity of the Laplace operator; which is nontrivial on general domains as far as I remember. This is why I would like a simple proof of the weaker statement that the domain is contained in $H^1$.
    $endgroup$
    – Nathanael Schilling
    9 hours ago





    $begingroup$
    I think I misunderstood the post above in that I thought it was a definition for $H^2$, whereas the definition was for $mathcal D(Delta)$ in terms of $H^2$. Nevertheless, my question still stands as the point I was asking about is contained in the reference to the ellipticity of the Laplace operator; which is nontrivial on general domains as far as I remember. This is why I would like a simple proof of the weaker statement that the domain is contained in $H^1$.
    $endgroup$
    – Nathanael Schilling
    9 hours ago













    $begingroup$
    In this situation, essential self-adjointness is exactly that the domain of the unique self-adjoint extension is $H^2$. (More generally, for semi-bounded operators, we can define analogous $H^2$, Friedrichs' extensions, and so on.)
    $endgroup$
    – paul garrett
    4 hours ago




    $begingroup$
    In this situation, essential self-adjointness is exactly that the domain of the unique self-adjoint extension is $H^2$. (More generally, for semi-bounded operators, we can define analogous $H^2$, Friedrichs' extensions, and so on.)
    $endgroup$
    – paul garrett
    4 hours ago











    1












    $begingroup$

    Maybe this is the argument you're looking for.



    Suppose you know that $Delta$ is essentially self-adjoint on $C^infty_c(mathbbR^n)$. This means $C^infty_c$ is a core for $Delta$, so for any $u in D(Delta)$, there is a sequence $u_n in C^infty_c$ such that $u_n to u$ and $Delta u_n to Delta u$ in $L^2$. In particular, we have
    $$|langle Delta(u_n - u_m), u_n - u_m rangle_L^2 | le |Delta u_n - Delta u_m|_L^2 |u_n - u_m|_L^2 to 0$$
    using Cauchy-Schwarz. On the other hand, since $u_n, u_m in C^infty_c$, we are perfectly justified in integrating by parts to say that $langle Delta(u_n - u_m), u_n - u_m rangle = |nabla (u_n - u_m)|^2_L^2$. So, since $|u_n - u_m|_L^2 to 0$ and $|nabla (u_n - u_m)|_L^2 to 0$, we have shown that $|u_n - u_m|_H^1 to 0$; in other words, $u_n$ is Cauchy in $H^1$. Now $H^1$ is a Hilbert space, so $u_n$ converges in $H^1$ to some $v in H^1$. But we already know $u_n to u$ in $L^2$, which is weaker than $H^1$ convergence, so we must have $u = v$, thus $u in H^1$.






    share|cite|improve this answer









    $endgroup$

















      1












      $begingroup$

      Maybe this is the argument you're looking for.



      Suppose you know that $Delta$ is essentially self-adjoint on $C^infty_c(mathbbR^n)$. This means $C^infty_c$ is a core for $Delta$, so for any $u in D(Delta)$, there is a sequence $u_n in C^infty_c$ such that $u_n to u$ and $Delta u_n to Delta u$ in $L^2$. In particular, we have
      $$|langle Delta(u_n - u_m), u_n - u_m rangle_L^2 | le |Delta u_n - Delta u_m|_L^2 |u_n - u_m|_L^2 to 0$$
      using Cauchy-Schwarz. On the other hand, since $u_n, u_m in C^infty_c$, we are perfectly justified in integrating by parts to say that $langle Delta(u_n - u_m), u_n - u_m rangle = |nabla (u_n - u_m)|^2_L^2$. So, since $|u_n - u_m|_L^2 to 0$ and $|nabla (u_n - u_m)|_L^2 to 0$, we have shown that $|u_n - u_m|_H^1 to 0$; in other words, $u_n$ is Cauchy in $H^1$. Now $H^1$ is a Hilbert space, so $u_n$ converges in $H^1$ to some $v in H^1$. But we already know $u_n to u$ in $L^2$, which is weaker than $H^1$ convergence, so we must have $u = v$, thus $u in H^1$.






      share|cite|improve this answer









      $endgroup$















        1












        1








        1





        $begingroup$

        Maybe this is the argument you're looking for.



        Suppose you know that $Delta$ is essentially self-adjoint on $C^infty_c(mathbbR^n)$. This means $C^infty_c$ is a core for $Delta$, so for any $u in D(Delta)$, there is a sequence $u_n in C^infty_c$ such that $u_n to u$ and $Delta u_n to Delta u$ in $L^2$. In particular, we have
        $$|langle Delta(u_n - u_m), u_n - u_m rangle_L^2 | le |Delta u_n - Delta u_m|_L^2 |u_n - u_m|_L^2 to 0$$
        using Cauchy-Schwarz. On the other hand, since $u_n, u_m in C^infty_c$, we are perfectly justified in integrating by parts to say that $langle Delta(u_n - u_m), u_n - u_m rangle = |nabla (u_n - u_m)|^2_L^2$. So, since $|u_n - u_m|_L^2 to 0$ and $|nabla (u_n - u_m)|_L^2 to 0$, we have shown that $|u_n - u_m|_H^1 to 0$; in other words, $u_n$ is Cauchy in $H^1$. Now $H^1$ is a Hilbert space, so $u_n$ converges in $H^1$ to some $v in H^1$. But we already know $u_n to u$ in $L^2$, which is weaker than $H^1$ convergence, so we must have $u = v$, thus $u in H^1$.






        share|cite|improve this answer









        $endgroup$



        Maybe this is the argument you're looking for.



        Suppose you know that $Delta$ is essentially self-adjoint on $C^infty_c(mathbbR^n)$. This means $C^infty_c$ is a core for $Delta$, so for any $u in D(Delta)$, there is a sequence $u_n in C^infty_c$ such that $u_n to u$ and $Delta u_n to Delta u$ in $L^2$. In particular, we have
        $$|langle Delta(u_n - u_m), u_n - u_m rangle_L^2 | le |Delta u_n - Delta u_m|_L^2 |u_n - u_m|_L^2 to 0$$
        using Cauchy-Schwarz. On the other hand, since $u_n, u_m in C^infty_c$, we are perfectly justified in integrating by parts to say that $langle Delta(u_n - u_m), u_n - u_m rangle = |nabla (u_n - u_m)|^2_L^2$. So, since $|u_n - u_m|_L^2 to 0$ and $|nabla (u_n - u_m)|_L^2 to 0$, we have shown that $|u_n - u_m|_H^1 to 0$; in other words, $u_n$ is Cauchy in $H^1$. Now $H^1$ is a Hilbert space, so $u_n$ converges in $H^1$ to some $v in H^1$. But we already know $u_n to u$ in $L^2$, which is weaker than $H^1$ convergence, so we must have $u = v$, thus $u in H^1$.







        share|cite|improve this answer












        share|cite|improve this answer



        share|cite|improve this answer










        answered 3 hours ago









        Nate EldredgeNate Eldredge

        20.9k3 gold badges74 silver badges119 bronze badges




        20.9k3 gold badges74 silver badges119 bronze badges



























            draft saved

            draft discarded
















































            Thanks for contributing an answer to MathOverflow!


            • Please be sure to answer the question. Provide details and share your research!

            But avoid


            • Asking for help, clarification, or responding to other answers.

            • Making statements based on opinion; back them up with references or personal experience.

            Use MathJax to format equations. MathJax reference.


            To learn more, see our tips on writing great answers.




            draft saved


            draft discarded














            StackExchange.ready(
            function ()
            StackExchange.openid.initPostLogin('.new-post-login', 'https%3a%2f%2fmathoverflow.net%2fquestions%2f335952%2fdomain-of-definition-of-laplace-operator-on-l2%23new-answer', 'question_page');

            );

            Post as a guest















            Required, but never shown





















































            Required, but never shown














            Required, but never shown












            Required, but never shown







            Required, but never shown

































            Required, but never shown














            Required, but never shown












            Required, but never shown







            Required, but never shown







            Popular posts from this blog

            Invision Community Contents History See also References External links Navigation menuProprietaryinvisioncommunity.comIPS Community ForumsIPS Community Forumsthis blog entry"License Changes, IP.Board 3.4, and the Future""Interview -- Matt Mecham of Ibforums""CEO Invision Power Board, Matt Mecham Is a Liar, Thief!"IPB License Explanation 1.3, 1.3.1, 2.0, and 2.1ArchivedSecurity Fixes, Updates And Enhancements For IPB 1.3.1Archived"New Demo Accounts - Invision Power Services"the original"New Default Skin"the original"Invision Power Board 3.0.0 and Applications Released"the original"Archived copy"the original"Perpetual licenses being done away with""Release Notes - Invision Power Services""Introducing: IPS Community Suite 4!"Invision Community Release Notes

            Canceling a color specificationRandomly assigning color to Graphics3D objects?Default color for Filling in Mathematica 9Coloring specific elements of sets with a prime modified order in an array plotHow to pick a color differing significantly from the colors already in a given color list?Detection of the text colorColor numbers based on their valueCan color schemes for use with ColorData include opacity specification?My dynamic color schemes

            Tom Holland Mục lục Đầu đời và giáo dục | Sự nghiệp | Cuộc sống cá nhân | Phim tham gia | Giải thưởng và đề cử | Chú thích | Liên kết ngoài | Trình đơn chuyển hướngProfile“Person Details for Thomas Stanley Holland, "England and Wales Birth Registration Index, 1837-2008" — FamilySearch.org”"Meet Tom Holland... the 16-year-old star of The Impossible""Schoolboy actor Tom Holland finds himself in Oscar contention for role in tsunami drama"“Naomi Watts on the Prince William and Harry's reaction to her film about the late Princess Diana”lưu trữ"Holland and Pflueger Are West End's Two New 'Billy Elliots'""I'm so envious of my son, the movie star! British writer Dominic Holland's spent 20 years trying to crack Hollywood - but he's been beaten to it by a very unlikely rival"“Richard and Margaret Povey of Jersey, Channel Islands, UK: Information about Thomas Stanley Holland”"Tom Holland to play Billy Elliot""New Billy Elliot leaving the garage"Billy Elliot the Musical - Tom Holland - Billy"A Tale of four Billys: Tom Holland""The Feel Good Factor""Thames Christian College schoolboys join Myleene Klass for The Feelgood Factor""Government launches £600,000 arts bursaries pilot""BILLY's Chapman, Holland, Gardner & Jackson-Keen Visit Prime Minister""Elton John 'blown away' by Billy Elliot fifth birthday" (video with John's interview and fragments of Holland's performance)"First News interviews Arrietty's Tom Holland"“33rd Critics' Circle Film Awards winners”“National Board of Review Current Awards”Bản gốc"Ron Howard Whaling Tale 'In The Heart Of The Sea' Casts Tom Holland"“'Spider-Man' Finds Tom Holland to Star as New Web-Slinger”lưu trữ“Captain America: Civil War (2016)”“Film Review: ‘Captain America: Civil War’”lưu trữ“‘Captain America: Civil War’ review: Choose your own avenger”lưu trữ“The Lost City of Z reviews”“Sony Pictures and Marvel Studios Find Their 'Spider-Man' Star and Director”“‘Mary Magdalene’, ‘Current War’ & ‘Wind River’ Get 2017 Release Dates From Weinstein”“Lionsgate Unleashing Daisy Ridley & Tom Holland Starrer ‘Chaos Walking’ In Cannes”“PTA's 'Master' Leads Chicago Film Critics Nominations, UPDATED: Houston and Indiana Critics Nominations”“Nominaciones Goya 2013 Telecinco Cinema – ENG”“Jameson Empire Film Awards: Martin Freeman wins best actor for performance in The Hobbit”“34th Annual Young Artist Awards”Bản gốc“Teen Choice Awards 2016—Captain America: Civil War Leads Second Wave of Nominations”“BAFTA Film Award Nominations: ‘La La Land’ Leads Race”“Saturn Awards Nominations 2017: 'Rogue One,' 'Walking Dead' Lead”Tom HollandTom HollandTom HollandTom Hollandmedia.gettyimages.comWorldCat Identities300279794no20130442900000 0004 0355 42791085670554170004732cb16706349t(data)XX5557367